If violations of any of a society's explicit rules routinely go unpunished, then that society's people will be left w...

Leila on August 21, 2013

Unpunished repetitive actions

Can someone please baby me through this I am not understanding the passage and answer choices ... :/ it's the only one in the section that I cannot wrap my head around !!! Thanx so much!!

Replies
Create a free account to read and take part in forum discussions.

Already have an account? log in

Melody on August 22, 2013

If rules are routinely unpunished, then there will be no moral guidance.

RU ==> not MG

MG ==> not RU

If you have no moral guidance, chaos will result

Not MG ==> C

Not C ==> MG

From these principles, the author concludes that a society should NEVER allow ANY of its rules to be broken with impunity.

This argument is flawed. The principles set forth lead to the conclusion that if routinely unpunished, then chaos. The correct conclusion is the prescription to avoid routinely un-punishing violations of the law. We are discussing a routine un-punishing, rather than just one instance.

(A) is incorrect because the argument does not do this. We never discuss what the conditions are for society to avoid chaos. Answer choice (A) is never discussed in the stimulus.

(B) is incorrect because it is irrelevant. Not addressing the purpose of the laws is not a flaw. Though he does fail to consider the reason behind the laws, this is not the logical fallacy.

(C) is incorrect because the stimulus never claims that the violation of some particular rules will lead to chaos.

(D) is CORRECT because it addresses the jump in tone from routine un-punishment to even one instance of un-punishment. The conclusion is that "a society ought never to allow any of its explicit rules to be broken with impunity." Any un-punishment is not the same thing as routine un-punishment.

(E) is incorrect because the stimulus does not do this. The concept of consequences due to broken rules was never discussed.

Hope that was helpful! Let me know if you have any other questions!

jing jing on April 14, 2020

Could you elaborate why C is incorrect? Thanks

Ben on April 26, 2020

Hi Jing Jing, thanks for the question.

While C may be tempting, it is incorrect because the conclusion of the argument is that society ought to not allow broken rules to ever go unpunished, but we do not know the exact reason for why that is.

Of course, it is based on the conditional and causal claims made above it. But we do not know that society shouldn't allow these rules to go unpunished purely because of ensuing chaos or because people will act in different ways. There is no support in one direction or the other.

It is important to realize that the real flaw with the argument is that chaos ensues if these rules are consistently left unpunished. Then the conclusion is about never letting a single rule breaking slide. These are two different things.

I hope this helps. Please let me know if you have any other questions!